当前位置:文档之家› 高中数学联赛之历年真题汇编(1981-2020)专题32函数、集合与复数(解析版)

高中数学联赛之历年真题汇编(1981-2020)专题32函数、集合与复数(解析版)

高中数学联赛之历年真题汇编(1981-2020)专题32函数、集合与复数(解析版)
高中数学联赛之历年真题汇编(1981-2020)专题32函数、集合与复数(解析版)

备战2021年高中数学联赛之历年真题汇编(1981-2020)

专题32函数、集合与复数

历年联赛真题汇编

1.【2020高中数学联赛B卷(第02试)】设集合A={1,2,?,19}.是否存在集合A的非空子集S1,S2,满足

(1) S1∩S2=?,S1∪S2=A;

(2) S1,S2都至少有4个元素;

(3) S1的所有元素的和等于S2的所有元素的乘积?

证明你的结论.

【答案】答案见解析

【解析】答案是肯定的.

设S2=1,2,x,y﹐2

所以2xy+x+y=187,

故(2x+1)(2y+1)=375=15×25,

所以x=7,y=12是一组解.

故取S1=3,4,5,6,7,8,10,11,13,14,15,16,17,18,19, S2=1,2,7,12,

则这样的S1,S2满足条件.

2.【2019高中数学联赛A卷(第02试)】设V是空间中2019个点构成的集合,其中任意四点不共面某些点之间连有线段,记E为这些线段构成的集合.试求最小的正整数n,满足条件:若E至少有n个元素,则E一定含有908个二元子集,其中每个二元子集中的两条线段有公共端点,且任意两个二元子集的交为空集.

【答案】2795

【解析】为了叙述方便,称一个图中的两条相邻的边构成一个“角”先证明一个引理:

]个两两无公共边的角(这里[a]表示实数a的整数部分).设G=(V,E)是一个简单图,且G是连通的,则G含有[|E|

2

引理的证明:对E的元素个数|E|归纳证明.

当|E|=0,1,2,3时,结论显然成立.

下面假设|E|≥4,并且结论在|E|较小时均成立.

只需证明,在G中可以选取两条边a、b构成一个角,在G中删去a、b这两条边后,剩下的图含有一个连通分支包含|E|-2条边.对这个连通分支应用归纳假设即得结论成立.

考虑G中的最长路P:v1v2?v k,其中v1,v2,?,v k是互不相同的顶点.因为G连通,故k≥3.

情形1:deg(v1)?2.由于P是最长路,v1的邻点均在v2,?,v k中,设v1v i∈E,其中3≤i≤k.则{v1v2,v1v i}是一个

角,在E 中删去这两条边.

若v 1处还有第三条边,则剩下的图是连通的;若v 1处仅有被删去的两条边,则v 1成为孤立点,其余顶点仍互相连通.总之在剩下的图中有一个连通分支含有|E |-2条边.

情形2:deg (v 1)=1,deg (v 2)=2.则{v 1v 2,v 2v 3}是一个角,在G 中删去这两条边后,v 1,v 2都成为孤立点,其余的点互相连通,因此有一个连通分支含有|E|?2条边.

情形3:deg (v 1)=1,deg (v 2)?3,且v 2与v 4,?,v k 中某个点相邻.则

是一个角,在G 中删

去这两条边后,v 1成为孤立点,其余点互相连通,因此有一个连通分支含有|E|?2条边.

情形4:deg (v 1)=1,deg (v 2)?3,且v 2与某个u ?{v 1,v 3,?,v k }相邻.由于P 是最长路,故u 的邻点均在v 2,?,v k 之中.因{v 1v 2,v 2u }是一个角,在G 中删去这两条边,则v 1是孤立点.

若处仅有边uv 2,则删去所述边后u 也是孤立点,而其余点互相连通.若u 处还有其他边uv i ,3≤i ≤k ,则删去所述边后,除v 1外其余点互相连通.总之,剩下的图中有一个连通分支含 有|E|?2条边. 引理获证.

回到原题,题中的V 和E 可看作一个图G =(V ,E ) 首先证明n ≥2795.

设V ={v 1,v 2,?,v 2019}.在v 1,v 2,?,v 61中,首先两两连边,再删去其中15条边(例如v 1v 2,v 1v 3,?,v 1v 16),共连

了C 612?15=1815条边,则这61个点构成的图是连通图.再将剩余的201-61=1958个点配成979对,每对两点

之间连一条边,则图G 中一共连了1815+979=2794条线段.由上述构造可见,G 中的任何一个角必须使用v 1,v 2,?,v 61相连的边,因此至多有[

18152

]=907个两两无公共边的角.故满足要求的n 不小于2795.

另一方面,若|E |≥2795,可任意删去若干条边,只考虑|E|=2795的情形.

设G 有k 个连通分支,分别有m 1,?,m k 个点,及e 1,?,e k 条边.下面证明e 1,?,e k 中至多有979个奇数.

反证法,假设e 1,?,e k 中有至少980个奇数由于e 1+?+e k =2795是奇数,故e 1,?,e k 中至少有981个奇数,k ≥981.不妨设e 1,e 2,?,e 981都是奇数,显然m 1,m 2,?,m 981?2.

令m =m 981+?+m k ?2,则有C m i 2?e i (1?i ?980),C m 2>e 981+?+e k ,

故2795=

∑e i

k i=1?

C m

2+∑

C m i 2980i=1

利用组合数的凸性,即对x ≥y ≥3,有C x 2+C y 2?C x+12+C y?12,可知当m 1,…,m 980,m 由980个2以及一个59构成时,C m 2+∑C m i

2980i=1

取得最大值. 于是C m

2+∑

C m i

2980i=1

?C 592+980C 22=2691<2795,

这与①矛盾.从而e1,?,e k中至多有979个奇数.

对每个连通分支应用引理,可知G中含有N个两两无公共边的角,

其中N=∑[e i

2]

k i=1?1

2

(∑e i

k

i=1

?979)=1

2

(2795?979)=908.

综上,所求最小的n是2795.

3.【2018高中数学联赛A卷(第02试)】设n、k、m是正整数,满足k≥2,且n?m<2k?1

k

n.设A是{1,2,…,m}的n元子集.

证明:区间(0,n

k?1

)中每个整数均可表示为a-a',其中a,a'∈A.

【答案】证明见解析

【解析】用反证法.假设存在整数x∈(0,n

k?1

)不可表示为a-a',a,a'∈A.作带余除法m=xq+r,其中0≤r

从而n=|A|?r?q+1

2?+(x?r)?q

2

?={

x?q+1

2

,2?q

x?q

2

+r,2|q

①.

这里?α?表示不小于α的最小整数.

由条件,我们有n>k

2k?1m=k

2k?1

(xq+r)②

又x∈(0,n

k?1

),故n>(k?1)x③

情形一q是奇数.则由①知,n?x?q+1

2

结合②,④可知,x?q+1

2?n>k

2k?1

(xq+r)?k

2k?1

xq,从而q<2k-1.

再由q是奇数可知,q≤2k-3,于是n?x?q+1

2

?(k?1)x,与③矛盾.

情形二q是偶数.则由①知,n?x?q

2

+r⑤

结合②,⑤可知,x?q

2+r?n>k

2k?1

(xq+r),从而xq

2(2k?1)

2k?1

r<(k?1)x

2k?1

,故q<2(k-1).

再由q是偶数可知,q≤2k-4,于是n?x?q

2

+r?(k?2)x+r<(k?1)x,

与③矛盾.

综上可知,反证法假设不成立,结论获证.

4.【2018高中数学联赛B卷(第02试)】设集合A={1,2,…,n},X、Y均为A的非空子集(允许X=Y).X中的最大元与Y中的最小元分别记为max X、min Y.求满足max X>min Y的有序集合对(X,Y)的数目.

【答案】22n ?2n (n +1)

【解析】先计算满足maxX ?minY 的有序集合对(X ,Y )的数目.对给定的m =maxX ,集合X 是集合{1,2,…,m -1}的任意一个子集与{m }的并,故共有2m?1种取法.

又minY ≥M ,故Y 是{m ,m +1,…,n }的任意一个非空子集,共有2n+1?m ?1种取法. 因此,满足maxX ?minY 的有序集合对(X ,Y )的数目是 (2n ?1)2?n ?2n +2n ?1=22n ?2n (n +1).

由于有序集合对(X ,Y )有(2n ?1)?(2n ?1)=(2n ?1)2个,于是满足maxX >minY 的有序集合对(X ,Y )的数目是(2n ?1)2?n ?2n +2n ?1=22n ?2n (n +1).

5.【2017高中数学联赛B 卷(第02试)】给定正整数m ,证明:存在正整数k ,使得可将正整数集N +分拆为k 个互不相交的子集A 1,A 2,?,A k ,每个子集A i 中均不存在4个数a 、b 、c 、d (可以相同),满足ab -cd =m . 【答案】证明见解析

【解析】取k =m +1,令A i ={x|x ≡i(?mod m +1),x ∈N +},i =1,2,…,m +1. 设a ,b ,c ,d ∈A i ,则ab ?cd ≡i ?i ?i ?i =0(?mod m +1),

故m +1|ab -cd ,而m +1?m ,所以在A 中不存在4个数a 、b 、c 、d ,满足ab ?cd =m .

6.【2015高中数学联赛(第02试)】设S ={A 1,A 2,?,A n }(n ?2),其中A 1,A 2,?,A n 为n 个互不相同的有限集合,满足对任意A i ,A j ∈S ,均有A i ∪A j ∈S .若k =min 1?i?n |A i |?2(|X |表示有限集合X 的元素个数),证明:存在x ∈∪A i n i=1,使得x 属于A 1,A 2,?,A n 中的至少n

k 个集合.

【答案】证明见解析

【解析】证法一证明更强的结论:对任意集合A i (1≤i ≤n ),存在x ∈A i ,使得x 属于A 1,A 2,?,A n 中的至少n

|A i

|个集

合.

若A 1,A 2,?,A n 与A i 的交集均不为空集,则根据平均值原理,知集合A i 中必存在某个元素x ,使得x 属于A 1,A 2,?,A n 中的至少n

|A i

|个集合.

若A 1,A 2,?,A n 中存在某些集合与A i 的交集为空集,不妨设这样的集合为A j 1,A j 2,?,A j i .则A i ∪A i j ,A i ∪A j 2,?,A i ∪A j i 互不相同,且均属于S.

于是,S 中其余n -2t -1个集合均与A i 的交集非空.

从而,集合A i 中所有元素在集合S 中出现的次数不少于k +tk +n ?2t ?1=n +(k ?2)t +k ?1>n . 故存在某个x ∈A i ,使得x 属于A 1,A 2,?,A n 中的至少n

|A i

|个集合.

综上,命题得证

证法二不妨设|A1|=k,|A1|?|A2|???|A n|,对A i(i=2,3,?,n)执行下列过程:

(1)先考虑A2.

若A2∩A1≠?,则将A2分为一组;

若A2∩A1=?,则将A2∪A1与A2两个集合分为一组.

(2)考虑A3.

若A3=A2∪A1,已经进行过分组.

若A3≠A2∪A1,再分两种情形考虑.

(i)A3∩A1≠?,则将A3分为一组;

(ii)A3∩A1=?,由A2≠A3,则A3∪A1≠A2∪A1.

此时,可将A3∪A1与A3两个集合分为一组.

7.【2014高中数学联赛(第02试)】设S={1,2,3,…,100},求最大的整数k,使得S有k个互不相同的非空子集,具有性质:对这k个子集中任意两个不同子集,若它们的交非空,则它们交集中的最小元素与这两个子集中的最大元素均不相同.

【答案】299?1

【解析】对有限非空实数集A,用minA与maxA分别表示A的最小元素与最大元素,考虑S的所有包含1且至少有两个元素的子集,一共299-1个,它们显然满足要求,

因为min(A i∩A j)=1

下面证明当k≥299时,不存在满足要求的k个子集.

我们用数学归纳法证明:对整数n≥3,在集合{1,2,…,n}的任意m(m≥2n-1)个不同非空子集A1,A2,…,A m 中,存在两个子集A i,A j,i≠j,满足A i∩A j≠?,min(A i∩A j)=maxA i①

显然只需对m=2n?1的情形证明上述结论.

当n=3时,将{1,2,3}的全部7个非空子集分成3组:第一组:{3},{1,3},{2,3};第二组:{2},{1,2};第三组:{1},{1,2,3}.由抽屉原理,任意4个非空子集必有两个在同一组中,取同组中的两个子集分别记为A i,A j,排在前面的记为A i,则满足式①

假设结论在n≥3时成立,考虑n+1的情形.若A1,A2,?,A2n,中至少有2n?1个子集不含n+1,对其中的2n?1个子集用归纳假设,可知存在两个子集满足式①.

若至多有2n?1?1个子集不含n+1,则至少有2n?1+1个子集含n+1,将其中2n?1+1个子集都去掉n+1,得到{1,2,…,n}的2n?1+1个子集.

由于{1,2,…,n}的全体子集可分成2n?1组,每组两个子集互补,故由抽屉原理,在上述2n?1+1个子集中一定有两个属于同一组,即互为补集.因此,相应

地有两个子集A i ,A j ,满足A i ∩A j ={n +1}. 这两个集合显然满足式①.故n +1时结论成立 综上所述,所求k max =299?1.

8.【2012高中数学联赛(第02试)】试证明:集合A ={2,22,?,2n ,?}满足: (1)对每个a ∈A 及b ∈N *,若b <2a -1,则b (b +1)一定不是2a 的倍数;

(2)对每个a ∈A (其中A 表示A 在N *中的补集),且a ≠1,必存在b ∈N *,b <2a -1,使b (b +1)是2a 的倍数. 【答案】证明见解析

【解析】(1)对于任意的a ∈A ,设a =2k ,k ∈N *,则2a =2k+1, 如果b 是任意一个小于2a -1的正整数,则b +1?2a ?1,

由于b 与b +1中,一个为奇数,它不含素因子2,另一个为偶数,它含素因子2的幂的次数最多为k ,因此,b (b +1)一定不是2a 的倍数.

(2)若a ∈A ,且a ≠1,设a =2k m ,其中h 为非负整数,m 为大于1的奇数.则2a =2k+1m . 下面给出三种证明方法:

证法一令b =mx,b +1=2k+1y ,消去b 得2k+1y ?mx =1,由于(2k+1,m )=1, 这方程必有整数解{x =x 0+2k+1t y =y 0+mt ,(其中t ∈Z ,(x 0,y 0)为方程的特解).

把最小的正整数解记为(x ?,y ?),则x ?<2k+1, 故b =mx ?<2a ?1,使b (b +1)是2a 的倍数.

证法二由于(2k+1,m )=1,由中国剩余定理知,同余方程组{x ≡0(?mod 2k+1)x ≡m ?1(?mod m)

在区间(0,2k+1m )上有解x =b ,

即存在b <2a -1,使b (b +1)是2a 的倍数.

证法三由于(2,m )=1,总存在r (r ∈N *,r ≤m -1),使2r ≡1(?mod m), 取t ∈N *,使tr >k +1,则2tr ≡1(?mod m).

存在b =(2tr ?1)?q ?2k+1m >0 (q ∈N),使0

9.【2011高中数学联赛(第02试)】证明:对任意整数n ≥4,存在一个n 次多项式f(x)=x n +a n?1x n?1+?+a 1x +a 0具有如下性质: (1)a 0,a 1,?,a n?1均为正整数;

(2)对任意正整数m 及任意k (k ≥2)个互不相同的正整数r 1,r 2,?,r k ,均有f(m)≠f (r 1)f (r 2)?f (r k ). 【答案】证明见解析

【解析】令f(x)=(x +1)(x +2)?(x +n)+2

将式①的右边展开即知f(x)是一个首项系数为1的正整数系数的n次多项式.

下面证明f(x)满足性质(2).

对任意整数t,由于n≥4,故连续的n个整数t+1,t+2,?,t+n中必有一个为4的倍数,从而由式①知f(t)≡2 (?mod?4),

因此,对任意k(k≥2)个正整数r1,r2,?,r k,有f(r1)f(r2)?f(r k)≡2k≡0(?mod?4),

但对任意正整数m,有f(m)≡2(?mod?4),

故f(m)≡f(r1)f(r2)?f(r k)(?mod?4),

从而f(m)≠f(r1)f(r2)?f(r k),

所以f(x)符合题设要求.

10.【2010高中数学联赛(第02试)】设k是给定的正整数,r=k+1

2

.记f(1)(x)=f(x)=x[x],f(l)(x)=f

(f(l?1)(x)),x∈R+,l≥2.证明:存在正整数m,使得f(m)(r)为一个整数.这里,[x]表示不小于实数x的最小整

数,例如[1

2

]=1,[1]=1.

【答案】证明见解析

【解析】记v2(n)表示正整数n所含的2的幂次.则当m=v2(k)+1时,f(m)(r)为整数.

下面我们对v2(k)=v用数学归纳法:

当v=0时,k为奇数,k+1为偶数,此时f(r)=(k+1

2)[k+1

2

]=(k+1

2

)(k+1)为整数.

假设命题对v-1(v≥1)成立.

对于v≥1,设k的二进制表示具有形式k=2v+αv+1?2v+1+αv+2?2v+2+?,这里,αi=0或者1,i=v+1,v+2,…

于是f(r)=(k+1

2)[k+1

2

]=(k+1

2

)(k+1)=1

2

+k

2

+k2+k

=

1

2

+2r?1+(αv+1+1)?2v+(αv+1+αv+2)?2v+1+?+22v+?

=k′+1

2

这里k′=2v?1+(αv+1+1)?2v+(αv+1+αv+2)?2v+1+?+22v+?,

显然k'中所含的2的幂次为v-1.故由归纳假设知r′=k′+1

2

经过f的v次迭代得到整数,由式①知,f(v+1)(r)是一个整数,这就完成了归纳证明. 11.【2006高中数学联赛(第02试)】解方程组

{

x?y+z?w=2

x2?y2+z2?w2=6 x3?y3+z3?w3=20 x4?y4+z4?w4=66

【答案】答案见解析

【解析】令p=x+z,q=xz,我们有

p2=x2+z2+2q,p3=x3+z3+3pq,p4=x4+z4+4p2q?2q2,

同样,令s=y+w,t=yw,

有s2=y2+w2+2t,s3=y3+w3+3st,s4=y4+w4+4s2t?2t2,

在此记号系统下,原方程组的第一个方程为p=s+2①

于是p2=s2+4s+4,p3=s3+6s2+12s+8,p4=s4+8s3+24s+32s+16,

现在将上面准备的p2,p3,p4和s2,s3,s4的表达式代入,得

x2+z2+2q=y2+w2+2t+4s+4,x3+z3+3pq=y3+w3+3st+6s2+12s+8,x4+z4+4p2q?2q2=y4+w4+4s2t?2t2+8s3+24s+32s+16.

利用原方程组的第二至四式化简,得q=t+2s?1②

pq=st+2s2+4s?4③

2p2q?q2=2s2t?t2+4s3+12s2+16s?25④

将式①和②代入式③,得t=s

2

?1⑤

将式⑤代入式②,得q=5

2

s?2⑥

将式①,⑤,⑥代入式④,得s=2.所以有t=0,p=4,q=3.

这样一来,x,z和y,w分别是方程X2-4X+3=0和Y2-2=0的两根,

即{x=3

z=1或{

x=1

z=3,

且{y=2

w=0或{

y=0

w=2

.

详言之,方程组有如下四组解

x=3,y=2,z=1,w=0或x=3,y=0,z=1,w=2,

或x=1,y=2,z=3,w=0,或x=1,y=0,z=3,w=2.

12.【2005高中数学联赛(第02试)】对每个正整数n,定义函数f(n)={

0,当n为平方数

[

{√n}

],当n不是平方数

,其中[x]表示

不超过x的最大整数,{x}=x-[x].试求:∑240

k=1

f(k)的值.【答案】768

【解析】对任意a ,k ∈N +,若k 2

设a =k 2+m (m =1,2,?,2k),√a =k +θ (0<θ<1), 则[

]=[

√a?k

]=[

√a+k a?k 2

]=[

2k+θm

],

因为0<2k+θm ?2k m

<1,

若在2k

m 与

2k+θm

之间存在整数t ,则2k m

2k+θm

于是,一方面2k

2k+θm

]=[2k

m

],所以∑

[

1

{a}]k 2

[2k

m

]2k

m=1,

于是∑(n+1)2

a=1

f(a)=∑

∑[2k i

]2k i=1

n

k=1

下面计算∑[2k i

]2k i=1

:画一张2k×2k 的表,第i 行中,凡是i 的倍数处填写“*”号,则这行的“*”号共[2k

i

]个,全表

的“*”号共∑

[2k

i

]2k i=1

个;

另一方面,按列收集“*”号数:第j 列中,若j 有T (j )个正因数,则该列便有T (j )个“*”号,故全表的“*”号个数共∑2k j=1

T(j)个,因此∑

[2k

i

]2k i=1

=∑2k j=1

T(j).

示例如下:

则∑(n+1)2a=1

f(a)=∑

∑2k j=1

T n k=1

(j)

=n[T(1)+T(2)]+(n ?1)[T(3)+T(4)]+?+[T(2n ?1)+T(2n)] ②

由此∑162

k=1

f(k)=∑(16?k)15

k=1[T(2k ?1)+T(2k)]

记a k =T(2k ?1)+T(2k) (k =1,2,?

,15),易知a k 的取值情况如下:

因此∑256k=1

f(k)=∑(16?k)15

k=1a k =783

由定义f(256)=f (162)=0,当k ∈{241,242,?,255}, 设k =152+r (16?r ?30),√k ?15=√152+r ?15=

√152+r+15

r 31

<

√152+r+15

<

r 30

,1?

30r

<

2<

31r

<2,

则[

{√k}

]=1 (k ∈{241,242,?,255})

从而∑240k=1

f(k)=783?∑256k=1

f(k)=783?15=768.

13.【2002高中数学联赛(第02试)】实数a ,b ,c 和正数λ,使得f(x)=x 3+ax 2+bx +c 有三个实数x 1,x 2,x 3,且满足: (i )x 1?x 2=λ; (ii)x 3>1

2(x 1+x 2).

2a 3+27c?9ab

λ3

的最大值.

【答案】3√32

【解析】由于f(x)=f(x)?f (x 3)=(x ?x 3)[x 2+(a +x 3)x +x 32+ax 3b ], 所经x 1,x 2是方程x 2+(a +x 3)x +x 32+ax 3+b =0的两个根,

由情形(i )可得(a +x 3)2?4(x 32+ax 3+b )=λ2,即3x 32+2ax 3+λ2+4b ?a 2=0,

再由情形(i )可得x 3=1

3

[?a +√4a 2?12b ?3λ]

且4a 2?12b ?3λ2?0

可以得到f(x)=x 3

+ax 2

+bx +c =(x +a 3

)3?(a 23

?b)(x +a 3)+

227

a 3+c ?1

3

ab ,

由f(x)=0可得1

3ab ?

227

a 3?c =(x 3+a 3)3

?(

a 23

?b)(x 3+a 3

)

由式①得x 3+a

3=13√4a 2?12b ?3λ2=

2√33

a 23

?b ?λ24

记p =

a 23

?b ,由式②和③可知p ?

λ24

且13

ab ?227

a 3?c =

2√39

√p ?

λ24

(p ?λ2),

令y =√p ?λ24

,则y ?0且13

ab ?

227

a 3?c =

2√39

y (y 2?3

4

λ2),

则y 3?

3λ24y +λ34

=y 3?

3λ24

y ?(λ2

)3

+

3λ2λ4

=(y ?λ2

)(y 2+λ

2

y +

λ24

?

3λ24

)=(y ?λ2

)2(y +λ)?0,

所以13

ab ?

227

a 3?c ??

√318

λ3

,于是2a 3+27c ?9ab ?

3√32

λ3

, 由此可得

2a 3+27c?9ab

λ3

?

3√32

取a =2√3,b =2,c =0,λ=2,

则f(x)=x 3+2√3x 2+2x 有根?√3?1,?√3+1,0, 显然满足假设条件,且2a 3+27c?9ab

λ3

=18

(48√3?36√3)=

3√32

综上所述

2a 3+27c?9ab

λ3

的最大值是3√32

.

14.【1999高中数学联赛(第02试)】给定实数a ,b ,c ,已知复数z 1,z 2,z 3满足{|z 1|=|z 2|=|z 3|=1

z 1z 2

+z 2z 3

+z 3z 1

=1 ,求|a

z 1+bz 2+cz 3|的值. 【答案】答案见解析 【解析】由已知z

1z 2+

z 2z 3

+

z 3z 1

∈R ,所以(z 1z 2

)+(z 2z 3

)+(z

3z 1

)=

z 1z 2

+

z 2z 3

+z

3z 1

.

即(z

1z 2

)+(z

2z 3

)+(z

3z 1

)=(z?

1z?2

)+(z?

2z?3

)+(z?

3z?1

),

又|z 1|=|z 2|=|z 3|=1,所以z?k =

1z k

(k =1,2,3),

代入式①得(z

1z 2

)+(z

2z 3

)+(z

3z 1

)=(z

2z 1

)+(z

3z 2

)+(z

1z 3

),

即z 12z 3+z 22z 1+z 32z 2=z 22z 3+z 32z 1+z 12z 2,

分解因式,得(z 1?z 2)(z 2?z 3)(z 3?z 1)=0. 所以z 1=z 2或z 2=z 3或z 3=z 1. 如果z 1=z 2,代入原式得

z 3z 1

=±i ,这时|az 1+bz 2+cz 3|=|z 1|?|a +b ±c i |=√(a +b)2+c 2,

类似地,如果z 2=z 3,则|az 1+bz 2+cz 3|=√(b +c)2+a 2, 如果z 3=z 1,则|az 1+bz 2+cz 3|=√(a +c)2+b 2.

15.【1997高中数学联赛(第02试)】试问:当且仅当实数x 0,x 1,?,x n (n ?2)满足什么条件时,存在实数

y 0,y 1,?,y n ,使得z 02=z 12+z 22+?+z n 2成立,其中z k =x k +i y k ,i 为虚数单位,k =0,1,…,n .证明你的结论.

【答案】答案见解析

【解析】易知题中式子等价于{∑x k 2n

k=1?x 02=∑y k 2n

k=1?y 0

2

∑x k n

k=1y k =x 0y 0 ①

若存在实数y 0,y 1,?,y n 使式①成立,则x 02y 02=(∑x k n k=1y k )2

, 由柯西不等式可得x 02y 02?(∑x k 2n

k=1)(∑y k 2n

k=1) ②

如果x 02>∑x k 2n

k=1,则由式①可得y 02>∑y k 2n

k=1, 从而x 02y 02>(∑x k 2n

k=1)(∑y k 2n

k=1).与式②矛盾, 于是得x 02?∑x k 2n k=1

反之,若式③成立,有两种情况:

(i )x 02=∑x k 2n

k=1,则取y k =x k ,k =0,1,2,?,n ,显然式①成立. (ii )x 02<∑x i 2n

k=1,记a 2=∑x k 2n

k=1?x 02>0,

从而x 1,?,x n 不全为0,不妨设x n ≠0,取y k =0,k =0,1,…,n -2,有 y n?1=

n

√x n?1+x n

,y n =

n?1

√x n?1+x n

.易知式①也成立.

综上可知,所求的条件为x 02?∑x k 2n

k=1.

16.【1994高中数学联赛(第02试)】x 的二次方程x 2+z 1x +z 2+m =0中,z 1,z 2,m 均是复数,且z 12?4z 2=

16+20i ,设这个方程的两个根α,β满足|α?β|=2√7,求|m|的最大值和最小值. 【答案】最大值是√41+7,|m|最小值是7?√41. 【解析】据表达定理有{

α+β=?z 1αβ=z 2+m

因为(α?β)2=(α+β)2?4αβ=z 12?4z 2?4m ,所以|α?β|2=|4m ?(z 12?4z 2)|=28,所以|m ?14

|(z 12

?4

z 2)=7.

即|m ?4+5i|=7,

这表明复数m 在以A (4,5)为圆点,以7为半径的圆周上,

又因为|OA|=√42+52=√41<7,故原点O在圆A内,联结OA,延长交圆A于两点B与C,

则|OB|=|OA|+|AB|=√41+7为|m|最大值.

|OC|=|CA|?|AO|=7?√41为|m|最小值.

所以|m|最大值是√41+7,|m|最小值是7?√41.

17.【1992高中数学联赛(第02试)】设集合S n={1,2,…,n}.若X是S n的子集,把X中的所有数的和称为X 的“容量”(规定空集的容量为0).若X的容量为奇(偶)数,则称X为S n的奇(偶)子集.

(1)求证:S n的奇子集与偶子集个数相等;

(2)求证:当n≥3时,S n的所有奇子集的容量之和与所有偶子集的容量之和相等.

(3)当n≥3时,求S n的所有奇子集的容量之和.

【答案】(1)证明见解析;(2)证明见解析;(3)答案见解析.

【解析】(1)设S n的奇子集的个数为a n,偶子集的个数为b n,则a n+b n=2n①

直接求a n,以[x]表示为不超过实数x的最大整数.

],l=n?k,从{2,4,…,2k}中任取一个子集(含空集)X1,

设k=[n

2

再从{1,3,…,2l-1}中任取一个含奇数个元素的子集X2,则X1与X2的并集便是一个奇子集,

反之,S n的任一奇子集可写成X1与X2之并.

(C l0+C l1+?+C l1)=2l?1(种)

X1的取法有2k种,X2的取法有C l1+C l3+?+C l2i?1=1

2

(2i-1是不大于l的最大奇数).

于是a n=2k?2l?1=2n?1.

由式①知b n=a n=2n?1.

(2)设A n(B n)表示S n中全体奇(偶)子集容量之和

(I)若n为奇数(n≥3),S n的所有奇子集可由下列两类子集组成:

(i)S n-1的奇子集;

(ii)S n-1的每一个偶子集与集{n}的并.

于是A n=A n?1+(B n?1+n?b n?1)=A n?1+B n?1+n?2n?2②

类似,可得B n=B n?1+(A n?1+n?a n?1)=A n?1+B n?1+n?2n?2③

得A n=B n.

(Ⅱ)若n是偶数(n≥4),S的所有奇子集可由下列两类子集组成:

(i)S n-1的所有奇子集;

(i)S n-1的每一个奇子集与集{n}的并.

于是A n =A n?1+(A n?1+n ?a n?1)=2A n?1+n ?2n?2 ④

类似,可得B n =2B n?1+n ?2n?2 ⑤

所以A n =B n .

综合情形(I )与(Ⅱ)的结论,对任何n ≥3,A n =B n .

(3)X 在S n 的余集记为X ?,则X 与X ?的容量之和等于S n 的容量, 即1+2+?+n =1

2n(n +1).

因此,S n 中所有子集的容量之和是2n?1?1

2

n(n +1)=2n?2?n(n +1).

因A n =B n ,

故A n =1

2?2n?2?n(n +1)=2n?3n(n +1)(n ≥3).

18.【1990高中数学联赛(第02试)】设E ={1,2,3,…,20},G ={a 1,a 2,a 3,?,a 100}?E ,且G 具有下列两条性质:

(1)对任何1≤i ≤j ≤100,恒有a i +a j ≠201; (2)∑a i 100i=1=10080;

试证:G 中的奇数的个数是4的倍数,且G 中所有数字的平方和为一个定数. 【答案】证明见解析

【解析】记αi =2i ?1,βi =201?a i (i =1,2,?,100). 令E i ={αi ,βi },则当i ≠j 时,E i ∩E j =?,且∪E i 100i=1=E . 由性质(1)知对任何1≤i ≤100,不能有E i ?G .

又由G 的元素个数恰等于集合E i 的个数,都是100, 因而G 必然恰只包含每个E i 中的一个元素.

现设G 中有k 个奇数,设对1?i i

t=1

+∑βj j≠i t

=10080

另一方面∑βj 100

j=1=2∑100j=i j =10100 ② 由式②-①,有∑(βi t ?αi t )k

t=1

=20

k ?201?2∑

αi t k t=1

=20,∑αi t k t=1

=12

(k ?201?20).

由式④首先推知k 必为正偶数,设k =2k ′

,则∑

αi i 2k ′k=1

=k ′?201?10.

注意到此式左端为偶数,从而k'必为正偶数,设k ′=2?k ′′,于是k =4k ′′.

这就证明了G中奇数的个数必为4的倍数.现在来计算G中数的平方和,以下运算中应用了式③.

∑αi2 100

i=1=∑αi

i

2

k

i=1

+∑βj2

k

j≠i i

=∑βj2

100

j=1

?∑βi

l

2

k

l=1

+∑αi

l

2

k

l=1

=∑(2j)2 100

j=1?∑(βi

i

+αi

i

)

k

i=1

(βi

i

?αi

i

)=4∑j2

100

j=1

?201×20

=4×100(100+1)(200+1)

6

?4020=1353400?4020=1349380.

19.【1983高中数学联赛(第02试)】函数f(x)在[0,1]上有定义,f(0)=f(1).如果对于任意不同的x1,x2∈[0,

1],都有|f(x2)?f(x1)|<|x2?x1|,求证|f(x2)?f(x1)|<1

2

.

【答案】证明见解析

【解析】不妨设0?x1

(1)如果x2?x1?1

2,则|f(x2)?f(x1)|<|x2?x1|?1

2

(2)如果x2?x1>1

2

,由f(0)=f(1)得

|f(x2)?f(x1)|=|f(x2)?f(1)+f(0)?f(x1)|?|f(x2)?f(1)|+|f(0)?f(x1)|

?(1?x2)+(x1?0)=1?(x2?x1)<1

2

.

所以|f(x2)?f(x1)|<1

2

(x1,x2∈[0,1]).

20.【1981高中数学联赛(第02试)】下列表中的对数值有两个是错误的,请予纠正:

【答案】答案见解析

【解析】先考虑表中x的取值:

0.021=3×7×10?3,

0.27=33×10?2,

1.5=3×2?1,

2. 8=22×7×10?1,

3,

5=2?1×10;

6=2×3,

7,

8=23,

9=32,

14=2×7.

其中0.27,3,9的对数仅与lg3有关,lg0.27=3lg3?2=6a?3b?2.

所以,推得lg3=2a?b,lg9=2lg3=4a?2b.

因此推得lg3=2a?b.

若上面三个对数有一个不正确,则三个数的对数值均错,与题设有两个错误矛盾.所以三个对数值均正确.

然后讨论仅与l g3,lg2有关的对数值,即真数分别为1.5,5,6,8的对数值lg1.5=lg3?lg2=3a?b+c.

所以lg2=?a?c,lg5=1?lg2=a+c.

则lg2=1?a?c,lg6=lg2+lg3=1+a?b?c,

所以lg2=1?a?c,lg8=3lg2=3?3a?3c,

因此lg2=1?a?c.

由上面讨论可见1.5的对数值表不正确.而其余三个数均正确,且由此作修正lg1.5=lg3?lg2=3a?b+c?1,lg0.021=lg3+lg7?3=2a+b+c?3.

所以lg7=2b+c,lg2.8=2?lg2+lg7?1=1?2a+2b?c,

所以lg7=2b+c,lg14=lg2+lg7=1?a+2b,

所以lg7=2b+c.

由上面讨论可见7的对数值表不正确.而其余三个数均正确,且作修正为表中lg7=2b+c.

优质模拟题强化训练

1.给定整数n(n≥3),记f(n)为集合{1,2,?,2n?1}的满足如下两个条件的子集A的元素个数的最小值:ⅰ.1∈A,2n?1∈A;ⅱ.子集A中的元素(除1外)均为A中的另两个(可以相同)元素的和.

(1)求f(3)的值;

(2)证明:f(100)≤108.

【答案】(1)f(3)=5(2)见解析

【解析】

1.设集合A?{1,2,?,23?1},且A满足(ⅰ)、(ⅱ).则1∈A,7∈A.

由于{1,m,7}(m=2,3,?,6)不满足(ⅱ),故|A|>3.

又{1,2,3,7},{1,2,4,7},{1,2,5,7},{1,2,6,7},{1,3,4,7},{1,3,5,7},{1,3,6,7},{1,4,5,7},{1,4,6,7},{1,5,6,7}

都不满足(ⅱ),故|A|>4.

而集合{1,2,4,6,7}满足(ⅰ)、(ⅱ),故f(3)=5.

2.首先证明:

f(n+1)≤f(n)+2(n=3,4,?).①

事实上,若A?{1,2,?,2n?1}满足(ⅰ)、(ⅱ),且集合A的元素个数为f(n).

令B=A∪{2n+1?2,2n+1?1}.

由2n+1?2>2n+1?1,知|B|=f(n)+2.

又2n+1?2=2(2n+1?1),2n+1?1=1+(2n+1?2),

则B?{1,2,?,2n+1?1},且集合B满足(ⅰ)、(ⅱ).

从而,f(n+1)≤|B|=f(n)+2.

其次证明:

f(2n)≤f(n)+n+1(n=3,4,?).②

事实上,设A?{1,2,?,2n?1}满足(ⅰ)、(ⅱ),且集合A的元素个数为f(n).令

B=A∪{2(2n?1),22(2n?1),?,2n(2n?1),22n?1}.

由2(2n?1)<22(2n?1)

则B?{1,2,?,22n?1},且|B|=f(n)+n+1.

而2k+1(2n?1)=2k(2n?1)+2k(2n?1)(k=0,1,?,n?1),

22n?1=2n(2n?1)+(2n?1),

则B满足(ⅰ)、(ⅱ).

从而,f(2n)≤|B|=f(n)+n+1.

由式,①、②得f(2n+1)≤f(n)+n+3.

反复利用式②、③得f(100)≤f(50)+51≤f(25)+26+51

≤f(12)+15+77≤f(6)+7+92

≤f(3)+4+99=108.

2.设X是有限集,t为正整数,F是包含t个子集的子集族:F={A1,A2,?,A t}.如果F中的部分子集构成的集族

S满足:对S中任意两个不相等的集合A、B,A?B,B?A均不成立,则称S为反链.设S1为包含集合最多的反链,S2是任意反链.证明:存在S2到S1的单射f,满足?A∈S2,f(A)?A或A?f(A)成立.

【答案】证明见解析

【解析】

记|S1|=r,称包含r个元素的反链为最大反链,最大反链可能不唯一

称F的子集P为链,如果?A,B∈P,A?B,B?A之一成立.

我们证明结论:F可以拆分为r个链P i(1?i?r)的并(即Dilworth定理).

对t进行归纳证明.t=1时显然成立.设命题对t-1成立,先假设存在一个最大反链S,使得F中既有集合真包含S 中的某个集合,也有集合是S中的某个集合的真子集.记前者的全体为F1,后者的全体为F2,即

F1={A i∈F|A i包含S中的某个集合},

F2={A i∈F|A i是S中的某个集合的子集},

则F1∪S,F2∪S均是F的真子集,从而由归纳假设可将F1∪S,F2∪S都可以拆成r个链的并.F1∪S中的链以S中的元素开始,F2∪S中的链以S中的元素结束.将这些链“接”起来就将F分成了r条链.

现在假设不存在这样的反链,从而每个最大反链要么满足F1=?,要么满足F2=?.前者意味着S中的子集都是“极大”子集(不是另一个A i的真子集),后者意味着S中的子集都是“极小”子集(不真包含另一个A i),从而至多有两个最大反链.如果极大子集构成的反链和极小子集构成的反链均为最大反链,则任取极大子集A,以及极小子集B?A,将A、B都去掉用归纳假设将剩下的集合拆分成r-1条链,再加上链B?A即可如果其中之一不是最大反链,不妨设极大子集构成的反链是唯一的极大反链,任意去掉一个极大子集归纳即可.结论证毕.

现在将F拆分成r条链,则每条链中恰有一个S1中的子集,且至多有一个S2中的子集.将每个S2中的子集对应到所在链中S1的元素,就得到了从S2到S1满足要求的映射.

3.证明对所有的正整数n≥4,存在一个集合S,满足如下条件:

(1)S由都小于2n?1的n个正整数组成;

(2)对S的任意两个不同的非空子集A、B,集合A中所有元素之和不等于集合B中所有元素之和.

【答案】见解析

【解析】

当n=4时,取S={3,5,6,7},则S满足条件.

其次,当n≥5时,令S={3,23,24,?,2n?2,2n?1?3,2n?1?2,2n?1?1}.

下面证明这样的S满足条件.

事实上,设A、B是S的两个不同的非空子集,

令f(X)表示集合X的所有元素之和,要证明的目标是f(A)≠f(B).

不妨设A∩B=?,注意到,对任意m∈N?均有1+2+4+?+2m?1=2m?1<2m.

所以,当a=2n?1?3,b=2n?1?2,c=2n?1?1都不属于A∪B时,均有f(A)≠f(B).

进一步,由于3+23+24+?+2n?2=2n?1?5,

所以当a、b、c中恰有一个属于A∪B时,例如a∈A,将有f(A)>f(B),此时f(A)≠f(B);

类似地讨论a、b、c中有两个或3个同时属于A∪B时,均可得出f(A)≠f(B).

综上所述,当n≥4时满足条件的S都存在.

4.设M是由有限个正整数构成的集合,且M=A1∪A2∪?∪A20=B1∪B2∪?∪B20,这里A i≠?,B i≠?,i =1,2,…,20.并对任意的1≤i

【解析】

记min{|A1|,|A2|,?,|A20|,|B1|,|B2|,?,|B20|}=t.

不妨设|A1|=t,A1∩B i≠?,i=1,2,...,k;A1∩B j=?,j=k+1,k+2, (20)

设a i∈A1∩B i,i=1,2,…,k.

因为对任意的1≤i

又对任意的1≤i≤20,1≤j≤20,若A i∩B j=?,则|A i∪B j|≥18,

所以当j=k+1,k+2,…,20时,|A1|+|B j|=|A1∪B j|≥18.

即,当j=k+1,k+2,…,20时,|B j|≥18?t.

所以|M|=|B1∪B2∪?∪B20|

=|B1|+|B2|+?+|B k|+|B k+1|+?+|B20|≥kt+(20?k)(18?t)

=360+2kt?18k?20t=180+2(k?10)(t?9).

若t≤9,则k≤t≤9,|M|=180+2(k?10)(t?9)≥180.

若t≥10,则|M|≥20t≥200.所以总有|M|≥180.

另一方面,取A i=B i={9(i?1)+1,9(i?1)+2,?,9(i?1)+9},其中i=1,2, (20)

则M=A1∪A2∪?∪A20=B1∪B2∪?B20={1,2,?,180}符合要求.

此时,|M|=180.

综上所述,集合M的元素个数的最小值为180.

5.已知n为正整数,集合I={1,2,?,n}(n>3)的k个三元子集A1,A2,…,A k满足:对任何I的其他三元子集B,均存在整数m和子集A i(1≤i≤k)使得{y|y≡x+m(modn),1≤y≤n,x∈A i}=B.求k的最小值.

【答案】k min=1+[n2?3n

6

]

【解析】

若a、b、c∈Z+,且a+b+c=p,

则称(a,b,c)是长为p的“循环组”,并约定(a,b,c)、(b,c,a)、(c,a,b)为同一个循环组.考虑长为n的循环组的数目.

a、b、、c中有两个相等的循环组有[n?1

2

]个;

若3|n,a、b、、c互不相等的循环组个数为1

3[C n?1

2?1?3([n?1

2

]?1)];

若n≡0(mod3),互不相等的循环组个数为1

3(C n?1

2?3[n?1

2

]).

综上,长为n的不同循环组的总个数为1+[n 2?3n

6

].

对于每个长为n的循环组(a,b,c),取集合I的一个三元子集B={1,a+1,a+b+1},存在一个子集A i与之对应,

且易验证不同的循环组对应的子集A i也不同,从而,k≥1+[n 2?3n

6

].

另一方面,对于前面的1+[n 2?3n

6

]个循环组中的每个(a,b,c),取与之对应的子集{1,1+a,1+a+b},共得到1

+[n2?3n

6

]个不同子集.

接下来说明这些子集满足要求.

事实上,对集合I的每个子集B={x1,x2,x3}(不妨设x1

令a=x2?x1,b=x3?x2,c=n?(x3?x1).

则得到一个长为n的循环组(a,b,c),

该循环组对应的子集A i满足存在整数m=x1?1(或x2?1或x3?1)使得{y|y≡x+m(mod n),1≤y≤n,x∈A i}=B.

综上,k

min =1+[n2?3n

6

].

6.求m的最大值,使得从一个n元集的子集中可以选出m个不同的子集A1,A2,…,A m,满足(A i∩A k)?A j对所有1≤i

【答案】2n

【解析】

不妨设此n元集为M={1,2,?,n}.

相关主题
文本预览